Putnam 1990/A3

Συντονιστές: Demetres, silouan

Άβαταρ μέλους
Demetres
Γενικός Συντονιστής
Δημοσιεύσεις: 8989
Εγγραφή: Δευ Ιαν 19, 2009 5:16 pm
Τοποθεσία: Λεμεσός/Πύλα
Επικοινωνία:

Putnam 1990/A3

#1

Μη αναγνωσμένη δημοσίευση από Demetres » Τετ Φεβ 14, 2018 3:14 pm

Δίνεται κυρτό πεντάγωνο του οποίου οι κορυφές (ανά τρεις μη συνευθειακές) έχουν ακέραιες συντεταγμένες. Να δειχθεί ότι το εμβαδόν του πενταγώνου είναι μεγαλύτερο ή ίσο από 5/2



Λέξεις Κλειδιά:
Άβαταρ μέλους
Διονύσιος Αδαμόπουλος
Δημοσιεύσεις: 807
Εγγραφή: Σάβ Μαρ 19, 2016 5:11 pm
Τοποθεσία: Πύργος Ηλείας

Re: Putnam 1990/A3

#2

Μη αναγνωσμένη δημοσίευση από Διονύσιος Αδαμόπουλος » Τετ Φεβ 14, 2018 10:26 pm

Έστω A_1A_2...A_5 το πεντάγωνό μας.

Από το Pick's theorem βλέπε εδώ έχουμε πως:

E+1=\dfrac{b}{2}+i, όπου E το εμβαδόν, b το πλήθος των σημείων με ακέραιες συντεταγμένες πάνω στην περίμετρο του πολυγώνου και i τα σημεία με ακέραιες συντεταγμένες εσωτερικά.

Θέλουμε λοιπόν \dfrac{b}{2}+i\geq \dfrac{7}{2}.

Έστω x_i, y_i οι συντεταγμένες του A_i. Από την αρχή της περιστεροφωλιάς παρατηρούμε πως τουλάχιστον 3 σημεία θα έχουν ισοϋπόλοιπες \pmod{2} τις x συντεταγμένες.

Πάλι από την αρχή της περιστεροφωλιάς, από τα y αυτών των σημείων τα 2 θα είναι ισοϋπόλοιπα \pmod{2}.

Συνοψίζοντας υπάρχουν A_i και A_j, έτσι ώστε x_i\equiv x_j \pmod{2} και y_i\equiv y_j \pmod{2}. Επομένως το σημείο με συντεταγμένες (\dfrac{x_i+x_j}{2} , \dfrac{y_i+y_j}{2}) έχει ακέραιες συντεταγμένες. Αφού αυτό το σημείο είναι μέσο των A_i και A_j, λόγω της κυρτότητας δεν θα είναι εξωτερικό σημείο. Διακρίνουμε τις περιπτώσεις:

α) Το σημείο αυτό είναι εσωτερικό. Τότε θα είναι i\geq 1 και προφανώς b\geq 5 (έχουμε τις 5 κορυφές). Άρα πράγματι \dfrac{b}{2}+i\geq \dfrac{7}{2}.

β) Το σημείο αυτό είναι στην περίμετρο. Τότε αν για την ώρα παραβλέψουμε το A_i, θα μας μένει ένα πεντάγωνο στο οποίο θα πρέπει επίσης να υπάρχουν δύο κορυφές των οποίο το μέσο να έχει ακέραιες συντεταγμένες. Αν είναι εσωτερικό του αρχικού μας πενταγώνου, θα είναι i\geq 1 και b\geq 6, οπότε θα ισχύει η ανισότητα. Αν είναι σημείο της περιμέτρου του αρχικού μας πενταγώνου, θα είναι b\geq 7 οπότε πάλι είμαστε βέβαιοι πως θα επαληθεύεται η ανισότητα.


Houston, we have a problem!
Άβαταρ μέλους
silouan
Επιμελητής
Δημοσιεύσεις: 1398
Εγγραφή: Τρί Ιαν 27, 2009 10:52 pm

Re: Putnam 1990/A3

#3

Μη αναγνωσμένη δημοσίευση από silouan » Πέμ Φεβ 15, 2018 1:58 am

To πρόβλημα έχει εμφανιστεί και εδώ: https://artofproblemsolving.com/communi ... 13p6775154
Αυτή είναι και η πηγή που το γνώριζα, δεν ήξερα ότι είχε πέσει και σε παλιό Putnam.


Σιλουανός Μπραζιτίκος
Απάντηση

Επιστροφή σε “Συνδυαστική - Επίπεδο Αρχιμήδη (Seniors)”

Μέλη σε σύνδεση

Μέλη σε αυτήν τη Δ. Συζήτηση: Δεν υπάρχουν εγγεγραμμένα μέλη και 2 επισκέπτες